68

A simple but fundamental question.


The "something" means the whole Universe (known and unknown), it could be represented as the reality version of the set of all sets, which is itself debated. It includes all the Multiverses and such.

A better version of the whole Universe could be described with a Grothendieck universe.

The "nothing" means an absence of the whole Universe, not a particular void or emptiness in the whole Universe but a nonexistent whole Universe.


  • 3
    Cf. The complete works of Heidegger...
    –  Tom Boardman
    Jul 21, 2011 at 14:04
  • 4
    @Tom: You could probably turn that comment into a very good answer... Jul 27, 2011 at 11:57
  • 6
    no offense, but when you find yourself asking "why is there something instead of nothing" you may enjoy adressing a psychologist for potential suicidal behaviour
    –  propaganda
    Jan 23, 2012 at 0:23
  • 1  
    This is a meaningless question in the sense of Carnap. –  Ron Maimon Apr 17 '12 at 6:00
  • 2
    'Nothing' might be a result of 'something'. There was always 'something' but this 'something' is not always the same. Sometimes it changes in to 'something' else. This means that the 'something else' is proceeded by its own nothingness You can project this little theory on to our own brain: The concept of 'nothingness' which is fabricated by the brain is nothing more than a result of the fact that it was not there a while ago when its particles looked different in time and space.
    –  Youss
    Apr 26, 2012 at 21:48
  • 4
    Neither Grothendieck universe nor the "set of all sets" has anything whatsoever to do with your original question, the first has a well defined meaning in mathematics, the second does not.
    –  orbifold
    Jul 3, 2012 at 20:17
  • 1
    I would say that OP must define what kind of answer he wants. (What kind of procedure can evaluate the answer, or smth like that.) Before it's done I call the question "meaningless" - just like Carnap (above). Or, in SE terms, "unclear" and "not useful". But can't downote it yet.
    –  zaa
    Feb 17, 2013 at 14:45
  • If there was the same amount of matter and antimatter in the Universe so that they could mutually annihilate together into absolutely nothing (this may require anti-energy, but let's pretend that's possible). Would you consider there is something in the Universe or just nothing unevenly distributed?
    –  Trylks
    Sep 19, 2013 at 9:40
  • I think something does not transform itself into nothing. Matter and antimatter as you call it may only transform into something else. The fact that we don't have the devices (yet) to detect this something else doesn't mean it is not there. Oct 6, 2013 at 15:11
  • 1
    The question is impossible, You've assumed that there is Something rather than Nothing, and in metaphysics when we do this we are dead in the water. It's this reification of objects that causes all the problems. The impossibility of answering this question should tell us something but not many want to hear it.
    –  user20253
    Mar 1, 2018 at 13:08
  • @PeterJ While this question may not have an answer (so this is why I still didn't validate any answer), I am very interested into what this should tell us according to you. Apr 29, 2018 at 22:37
  • @GeoffroyCALA - Apologies, my comment was poor. I was suggesting that it is only when we assume that matter is naively real that this question becomes impossible, There are some philosophies that state that nothing really exists and for them this question would be a misunderstanding. A prior question would be 'Does the universe exist and in what sense'. As it stands I cannot answer the question because I disagree with its assumptions. A non-pre-emptive question that works would be, 'Why does there appear to be something rather than nothing'. This would allow a different kind of answer. . .
    –  user20253
    Apr 30, 2018 at 11:38
  • @PeterJ Would that answer be really that different ? Anyways I would still be interested by the answer of the modified question. May 27, 2018 at 17:20
  • does the question make sense? i'm not sure, i guess it depends what sort of answer you're looking for. just because a physicist can't help doesn't mean a philosopher should answer it. anyway there's this which is fairly unhelpful. it seems to say that this is not an empty world. why expect anything else? what are you actually asking?
    –  user38026
    Apr 21, 2019 at 10:41
  • @TomBoardman do you think that Heidegger is explaining why there's something rather than nothing in a way that knowledge of why can be verbalized (whether or not he succeeds)?
    –  user38026
    Apr 21, 2019 at 12:25

29 Answers

29

We don't know.

There are some very valiant attempts to engage the question here, and many of them even explore concepts well worth exploring. But just because we live in such a complex, information-packed age doesn't mean we need to pretend we know things we don't. The oracle at Delphi said that Socrates was the wisest man in Athens simply because he realised that he knew nothing, and supposedly that one statement is responsible for the existence of Western philosophy in the first place. She had a pretty good point.

I've never heard a good reason offered for why there is a universe/multiverse instead of there not being one. I may one day hear such a reason, but I couldn't begin to imagine how it might proceed. Nor can I think of an ultimate reason why "me" began. This is contrast, of course, to the scientific observations and models I can build of how a big bang might have lead to stars, galaxies and planets, a human being might have evolved on one of them, how physics could give rise to consciousness, and so on. But as to why that entire business is busily, well, businessing--a complete mystery. Sometimes the most honest thing you can do is admit that. Is it really so hard to do?

  • 1
    A good answer. This is similar to anon's answer but in layman's terms. But even if we don't/can't know with 100% certainty, maybe we can list several possibilities? That's how I imagine the ideal answer. Aug 23, 2011 at 19:36
  • 1
    Actually, a great answer. Most of the other answers here are really interesting and insightfull, but they don't tackle the question. The question lies in (or maybe outside) the limits of human reasoning, and simply trying to keep everything inside is not an answer.
    –  Koeng
    Oct 17, 2012 at 15:16
  • Matter and energy can not be created or destroyed according to Physics laws so assuming this can be applied back through ALL history does this imply there has always been matter and energy? So there has always been something....
    –  user128932
    Jul 13, 2014 at 7:09
  • Every explanation for why something has to exist would have to start with the assumption that something exists - we can't very well find prior principles to derive that fact from.
    –  Brilliand
    Jul 14, 2014 at 18:06
  • Actually, as Descartes pointed out, the argument itself does suffice for that. It's just that even taking the existence of something into account, there's no sufficient explanation for why that something exists.
    –  eMansipater
    Oct 25, 2015 at 9:36
  • Doesn't the anthropic principle play a role here? That is, if there were an alternate reality where there was nothing, there would simply be no-one to observe it; the only reality in which one can even ask the question is one in which there is something. I feel as though this and other answers have blown past this, or perhaps I'm not reading them deeply enough. This doesn't negate the truth of this answer nor even begin to address why things are as they are and not some other way. But being that nothing can have no observers, this is akin to asking why one is born alive and not born dead.
    –  Sputnik
    May 30, 2018 at 15:07
23

Before anything else one should be aware of the instrument used to answer questions. That instrument is human language. While there is no guarantee that such a system of patterns is powerful and expressive enough to reason about the necessity of all that is, we can still examine what can we reasonably say and understand about this.

Let us begin by finding out the difference between Something and Nothing.

Obviously they are opposites - first word denotes the present state of affairs of all that is and the other one its negation. We can be fairly confident that right now Something is and Nothing is not.

So why is it this way? Could there be Nothing instead of Something? Turns out the answer is surprisingly simple and straightforward: if Nothing could be, then it would inevitably be Something - giving us a contradiction with our initial premises, thus demonstrating that Nothing cannot be.

In other words, there is Something because it cannot be otherwise - it is an inevitability that arises from the complete, utter and total incapacity of Nothing to exist as such.

A more formal version of the same argument can be found from @anon's answer.

In short: Something is because Nothing is not.

Or to quote "The Way Of Truth" by Parmenides: For never shall this prevail, that things that are not are.


  • An update in response to comment by OP

This is not circular reasoning. You are missing the point that we reason only through language which is a limited system of rules. Breaking a rule results in error.

The above question makes the error of assuming that non-being can be. It is self-contradictory. In the same way one can ask why is there no darkness in light or why is there no silence in sound.

Apparently you presume that if there is a word ("nothing"), then it must have an object, i.e. a correspondence with something beyond your own imagination. Not true!

Nothing is the opposite of Something, it does not and can not be.

  • Thank you. Your answer is an expanded version of Leo's. But then one could ask "why nothingness does not exist?", and respond: "because something is". Similarly "why does chlorophyll is green instead of blue?" with the same principle one could answer: "because it isn't blue". Isn't this some kind of circular reasoning? Aug 6, 2011 at 14:52
  • @Geoffroy CALA: See the updated answer.
    –  Saul
    Aug 6, 2011 at 16:54
  • 2
    @Saul: excellent answer, especially the addendum. Aug 7, 2011 at 1:48
  • OK, that is a good argument. So in short, the question is invalid because obviously nothing isn't in our Universe so we can't even imagine a case where nothing could be. This question then may be invalid because of 2 interesting arguments: yours and Leo's (only something is, nothingness isn't) and ChrisEve's (completed by some of my ideas) neither something nor nothing really exist, our Universe has a real density and has not an infinite density or zero density. Those 2 arguments are based on the fragile assumption that true nothingness is impossible. Aug 7, 2011 at 11:06
  • The question doesn't at all assume that non-being can be, because non-being isn't. Nothingness is not a state of affairs but an absence of any such thing. When you say that being may be necessary (by virtue of nothingness being potentially impossible), you assume a cause which would make it necessary, and so you've inadvertently concluded that a first cause must exist).
    –  danielm
    Oct 13, 2012 at 9:09
  • But even if we are to assume that something always was, or is, the question still holds. It inquires about what the cause of being is, not in the temporal sense which may concern a sequential, mechanical event in time ("what caused the billiard ball to move?"), but precisely by what metaphysical cause a thing is. Unfortunately, modern ideas about causes have been castrated and constrained to a narrow sliver of what they once were, as Gilson rightly laments in his works. And with respect to Parmenides, he denied change of any kind. And your assertions about language undermine reason per se.
    –  danielm
    Oct 13, 2012 at 9:12
  • @danielm - Of course the question assumes that non-being (non-existense) can be. Look at it. It asks "Why is there something instead of nothing?". Ontologically, nothingness can not be. If nothing could be, then nothing would be something which is a contradiction of premises. That is logically equivalent to asking why is someone not dead when he is alive. Such questions are self-contradictory and thus meaningless. When it comes to Parmenides then no sane person writes a philosophy book while believing it to have no consequence. His "Way Of Truth" describes laws, not manifestation.
    –  Saul
    Oct 14, 2012 at 14:14
  • 12
    @Saul - You're playing word games. The question doesn't assume nothing is something in any meaningful way. The word "nothing" is indeed a noun, but it is understood by every sane person to be a mere placeholder. If that bothers you, then cut away the last three words of the question and we're off to the main argument. I don't understand the remainder of your response. Perhaps we've misunderstood one another. True nothingness is not impossible. It is only impossible for true nothingness to have been, so to speak. Thus we come to the question of the first (metaphysical) cause that always was.
    –  danielm
    Oct 14, 2012 at 22:53
  • @danielm - Not word games, but insistence on consistency and sound reasoning. Something is because it cannot not be; Nothing is not because it cannot be. From viewpoint of Parmenides, the nature of first cause is different - "How could what is perish? How could it have come to be? For if it came into being, it is not; nor is it if ever it is going to be. Thus coming into being is extinguished, and destruction unknown.". Hence, something is because it has always existed, be it first cause with subsequent causes, nirvana or ever-continuing cycles of creation and destruction.
    –  Saul
    Oct 15, 2012 at 12:59
  • 1
    @Saul"Your question makes the error of assuming that non-being can be. It's self-contradictory. Analogously absurd would be to ask why is there no darkness in light or why is there no silence in sound." You presume that non-being cannot be because...? Saying such nonsense as "Being is" is not self-referential, right? Or, seeing that it is, has no sense. So, "being is" is nonsense, which is another way of saying that the phrase has no meaning, no sense, and is referring to nothing. But wait, I thought saying "non-being [is]" is self-contradictory/ But, if "being is" refers to nothing, then
    –  Jon
    Oct 16, 2012 at 18:03
  • surely "non-being is" refers to something. But then again, "being is" refers to everything, which means that being is an "object," right? (It's also a nominalization!) Or, is it more like the words "all" or "everything" or the "universe" as it properly refers to everything? But, isn't is, again, just referring to itself. So, the phrase "being is" and, which means something like "all that is exists" looks pretty much like a tautology to me. How much sense do tautologies have? Wittgestein, in §215-6 of On Certainty helps shed some light on these sorts of "questions": "the idea of agreement
    –  Jon
    Oct 16, 2012 at 18:14
  • with reality' does not have “any clear application” (OC,§215). He explicates what he means: "The proposition 'it is written'" (OC, §216). Obviously, the "proposition 'it is written'" is an empirical proposition; however, because it is also a tautology, it's nonsensical and conveys nothing foundational or meaningful.
    –  Jon
    Oct 16, 2012 at 18:21
  • 1
    @Jon - Something and Nothing are opposites. Something is but Nothing is not. If Nothing could be, then it would inevitably be Something - giving us a contradiction with our initial premises, thus showing that Nothing cannot be. Which part of this are you failing to understand?
    –  Saul
    Oct 17, 2012 at 6:13
  • 6
    You nailed the issue: language. But that is exactly what is holding you from tackling the question. You are attached to the strutcture of the language, ignoring the meaning of the question. Simply rephrase: "There is something. Why is that the case?"
    –  Koeng
    Oct 17, 2012 at 15:04
  • @Saul. It looks to me as though you're trying to "do" some elementary ontology. Perhaps, looking at the actual use and meaning of the words you're using isn't the solution. Okay. Perhaps, we should look for other ways of describing a sentence like "something is...." You see: I find it difficult to find any meaning in such an odd (incomplete, and contextless) phrase. It's all so clear: "something is," "nothing is not." But, since we're talking about "opposites," isn't it a semantic issue? Or, is it a syntactical issue? No, I presume that you're assuming that meaningless constructions like ->
    –  Jon
    Oct 17, 2012 at 15:46
  • "something is" has sense taken out of context. The funny thing is that, outside of certain lazy ways of "doing" philosophy, these sort of sentences do not occur in everyday conversation. So, doesn't it become an issue of semantics? Your "premise" depends on quite a myopic understanding of meaning. That is the issue. My original comments were not intended to argue with what you were saying; but, they were intended to suggest that your answer may be meaningless; and, in many ways, it abuses the actual use of language by misusing simple words for (perhaps?) ontological purposes.
    –  Jon
    Oct 17, 2012 at 15:58
  • @Koeng - One cannot communicate an answer without adhering to a structure. Sure, a structure can be redefined, modified and so on but meaningful communication without a shared structure is not possible. To answer Your rephrased question: there is Something because it cannot be otherwise - it is an inevitability that arises from the complete, utter and total incapacity of nothingness to exist.
    –  Saul
    Oct 17, 2012 at 16:08
  • @Jon - Perhaps, perhaps not. I suspect our differences come from different implicit assumptions. Could you describe, in the most terse way you can express it, what is Nothing?
    –  Saul
    Oct 17, 2012 at 16:10
  • @Saul I understand and agree with the first part. What I mean is that all you are saying is within the limits of our current tools of logic and language. The question itself is not meaningless, it is only meaningless if you try to bound it by those tools. They are great, useful tools, but you can't dismiss the question by saying it doesn't fit our tools. The question leads to a close examination of our limits and maybe to a development of better tools. Maybe our brains (or any brain) can never really answer that question, and that's ok. But that doesn't mean it is meaningless.
    –  Koeng
    Oct 17, 2012 at 16:26
  • 2
    @Saul To address the more logical part of your comment, you state that "there is Something because it cannot be otherwise - it is an inevitability that arises from the complete, utter and total incapacity of nothingness to exist". But for that you assume that the existence of reality is reflected in logic. Yes, if we assume that, you are right. But what reason do we have to assume that the ultimate existence of reality is defined by what we understand as logical? Did logic exist before the reality bounds to it and say "well, if it must be non-contradictory, then I must also exist"?
    –  Koeng
    Oct 17, 2012 at 16:31
  • @Saul. Concepts are blurred. They're neither rigid nor fixed. And, for us, "nothing" has varied but often interconnected meanings depending on the circumstances the word is used in and meaning trying to be conveyed. However, the Parmenidean idea of conservation is much more similar to Lavoisier's conservation of mass or the 1st Law of thermodynamics (if they're both understood as formulations of mass/energy conservation--i.e. matter can change into massless energy). Existentially, nothing's place lies within physics. But physically speaking, we're still looking for it. As a word that I use in
    –  Jon
    Oct 17, 2012 at 16:52
  • in everyday conversation, it is quite apparent that the meaning of what I am saying is not abstract or metaphysical. "I'm feeling rather lazy today, I just feel like doing nothing." I've yet to find someone who couldn't grasp my meaning.
    –  Jon
    Oct 17, 2012 at 16:54
  • @Koeng - If nature of reality is not reflected in logic then we cannot prove or discover that it is so. There is a famous Zen story, ending with: "Truth has nothing to do with words. Truth can be likened to the bright moon in the sky. Words, in this case, can be likened to a finger. The finger can point to the moon's location. However, the finger is not the moon. To see the moon, it is necessary to gaze beyond.". So, in a sense, this answer is a doorway but doorways come in different sizes and shapes, depending on the carpenter and the tools.
    –  Saul
    Oct 17, 2012 at 16:56
  • @Jon - Vagueness in understanding a concept on your own part is not a valid reason for declaring that concept meaningless. Try again, describe, what is Nothing? How is it different from Something?
    –  Saul
    Oct 17, 2012 at 17:08
  • @Saul. I have trouble with the statement that "if nature of reality is not reflected in logic then we cannot prove or discover that it is so." One of my professors once said something to this effect: if you expect the sciences to be logical (it was a quantum physics course), you're in the wrong major. That is not to say, however, that we cannot make sense of our observations––only that our understanding of reality need not be logical. I realize that begs the question of what "logical" means here. And so it does.
    –  Jon
    Oct 17, 2012 at 17:09
  • @Saul. Okay, now I see your issue. I'm circling around questions that concern meaning; and, in doing so, I'm trying to show that the apparent "vagueness" of a concept comes about when the context is not stated or clear (the philosopher, searching for what it means "to know" something, repeats over and over to himself in the garden, "I know that's a tree"). In cases where the meaning of the phrase is clear, there is no ambiguity. It only becomes vague when certain people take a word out of context, and pretend as though there is a meaning attached to a word, like some sort of entity.
    –  Jon
    Oct 17, 2012 at 17:23
  • See the first 131 sections of the Philosophical Investigations for a more thorough and thought-provoking inquiry. And, if you find the method at all convincing and are still interested in Parmenides's conservation of being, I'd recommend looking into Paul Feyerabend's Conquest of Abundance (esp. his essay, "Parmenides and the Logic of Being".) If you are interested in how concepts change (esp. in science), you may find Against Method or Thomas Kuhn's Structure of Scientific Revolutions intriguing.
    –  Jon
    Oct 17, 2012 at 17:29
  • @Jon - Your trouble comes from not understanding what your professor meant. Logic is a science of valid reasoning and it is useful because invalid reasoning leads to everything but the truth. However one cannot expect to aquire deeper understanding without a serious effort. Hence, try again, what is Nothing? How is it different from Something?
    –  Saul
    Oct 17, 2012 at 17:30
  • Or, if you are interested in the relation between language and "reality," check out Progress in Colour Studies: Volume II. Psychological Aspects. The essay, “Colour Categories and Category Acquisition In Himba and English" directly addresses the relation between perceived color categories and the acquisition color words.
    –  Jon
    Oct 17, 2012 at 17:31
  • @Saul. Well, no: actually, the context of the anecdote concerned the relation between the wave function and the "reality" (the things) to which it seems to refer. The "truth" was not at issue. For example, it was more along the lines of what Heisenberg meant when he said that quantum theory is "a wonderful example [of symbolic scientific description] for it shows that one can clearly understand a state of affairs yet know that one can describe it only in images and smilies."
    –  Jon
    Oct 17, 2012 at 17:45
  • @Jon - Thanks, but I am not much of a fan of modern philosophers. With a few exceptions (such as Kuhn and Wittgenstein), they mostly tend to be in the business of reinventing the wheel in a much worse way. Less is more, especially in philosophy.
    –  Saul
    Oct 17, 2012 at 17:51
  • The question, "what, then, is nothing" is nonsense. Not to be peevish, but the reason you're dissatisfied with my replies is because in giving various ways in which "nothing" is used, the blurred edges of the concepts are becoming clearer. And, in fact, while you think the responses are vague, the meaning of the sentences, the phrases, and the ideas in which the word "nothing" occurs are clear to anyone who speaks English. At least to me, that would mean that there is "no one meaning of a word," and, in fact, words need contexts to mean anything. But the simple question, "what is nothing" has
    –  Jon
    Oct 17, 2012 at 17:55
  • no context, and thus, has no sense. Or, in other words, appearing to be a deep question, it's forgotten its place in language, in conversation, in thought. Thus, its depth becomes a sort of playful shallowness. But these sorts of questions do not just "play with words," but they also ignore the way people actually speak and convey meaning.
    –  Jon
    Oct 17, 2012 at 18:03
  • @Jon - If the question "what, then, is nothing" is nonsense then the original question "Why is there something instead of nothing?" becomes equal to "Why is there something instead of {nonsense}?". However one cannot expect to have a meaningful answer to a question that has nonsensical parts. If we follow your reasoning, we are left to conclude that either: (a) this whole question is meaningless; (b) "what, then, is nothing" must have a sensible answer. Try again, this time the other way around, how is Something different from Nothing?
    –  Saul
    Oct 17, 2012 at 18:13
  • @Saul. I find Feyerabend is interesting, as he's less of a dogmatist than Kuhn. Kuhn credits Feyerabend with introducing him to Witt., but then became a pretty ardent relativist. Feyerabend offers some incisive arguments against relativists and realists. But, as for older philosophers, Xenophanes was one of the first thinkers to stress the importance of objects' circumstance rather than their intrinsic natures. Aristotle loved words that had different meanings in different circumstances, and knew well that these meanings depended on their circumstances.
    –  Jon
    Oct 17, 2012 at 18:16
  • Why not just ask, "what is something?" Or, put in a more general way, ask, "what is anything?" I assume you might have qualms with this reformulation (although I see it as fitting); but, if there are problems with it, what are they? This is not an attempt to dodge the question, but to illuminate an aspect of the question, "what is something," as one would presumably began from that starting point before the question of difference is considered.
    –  Jon
    Oct 17, 2012 at 18:23
  • @Jon - You suggested that this answer might not be meaningful but so far you have not demonstrated how you reached that conjecture. To get there, one must have concepts both about Something and Nothing. For me, Something is a generic term referring to the present state of affairs in their totality (all laws of nature both discovered and undiscovered, matter, energy fields and so on) and Nothing is a generic term referring to its complete negation, a complete absense of Something. Which concepts did you apply to reach the conjecture that this answer might not be meaningful?
    –  Saul
    Oct 17, 2012 at 18:40
  • @Saul. The point of the reformulation was to show the inadequacy of looking for the meaning of a word without a context in which the phrase or sentence that contains the words has sense. What are the differences between the sentences "there is something over wall" and "there is something?" Well, I could expand the first formulation and give reasons for my thinking that there's something on the other side of the wall (a noise, a shadow, etc.); but, "there is something" doesn't denote a specific entity--but, as you mentioned, you're taking "something" to refer to everything known and unknown.
    –  Jon
    Oct 17, 2012 at 19:08
  • And, as “anything” is used to refer to a thing (often an unspecified thing—which would include hitherto unknown laws of nature), “why is there anything” would be the general form of the question, “why is there something.” To put it another way, in terms of the meaning of the word, (see also J.L. Austin, in his essay, the “Meaning of a Word” for a more complete analysis of this sort of problem), the general form of the question, “what is the meaning of a word” becomes “what is the meaning of any word,” and thus, the questioning leads into nonsense.
    –  Jon
    Oct 17, 2012 at 19:08
  • 1
    Please consider taking these great conversations to chat, guys. Comments are really just for asking clarifying questions. You should be seeing some automated warnings and a helper link that will push these comments into a chatlog. I encourage you to do this; at some point, we may well purge these sorts of enormous chat-like discussions in comments.
    –  Joseph Weissman
    Oct 17, 2012 at 19:25
  • But, it is also worth pointing out that anything is quite often used with a negation: "I'm not talking about anything, I'm talking this thing, or something specific" (i.e. anything can be used as the general form of formulating a question about an abstract something; and, negating anything in a phrase is often equivalent to referring to something). But, as nothing and something are "opposites," it would seem that both the negations of something and anything are nothing; but the negation of anything was something (and now also nothing). See: the meaning is context dependent.
    –  Jon
    Oct 17, 2012 at 19:26
  • @JosephWeissman - Thanks for the reminder. It is just that comments are a much better medium for discussion as chat is instantaneous and there is no time to think properly.
    –  Saul
    Oct 17, 2012 at 19:39
  • @JosephWeissman. I agree with Saul; it also helps to have all the answers and questions on the same page so as to provide a fuller context. Though, I did receive an error message when trying clicking on the link after your post.
    –  Jon
    Oct 17, 2012 at 19:43
  • @Jon - I see that your main goal is to encourage me into improving this answer, perhaps I will. On the meaning of words my view is that any human language is a system for transmitting sensation or awareness. There is a sender with certain awareness or sensation, a message where he encodes it into, a medium for transmission and a receiver who may or may not succeed in reconstructing the transmitted awareness. As such, there is no nonsense, only vague awareness or inefficient (de)coding or a lossy medium. Think people understanding each other without words, I am sure you have experienced it.
    –  Saul
    Oct 17, 2012 at 19:52
  • @Saul. One of the reasons I found Progress in Colour Studies intriguing is because, in it, the researchers compare learning color categories for English and Himba children: “The advantage for items central (focal) to children’s native language categories also increased throughout the longitudinal study. Thus, the importance that Rosch gave to focality in establishing categories seems justified from the present data; nevertheless, it is important to stress that the focality is not universal but, as shown both at first testing and longitudinally, it is language dependent" (p.159-73).
    –  Jon
    Oct 17, 2012 at 20:23
  • I would add, perhaps as a closing comment (as to keep Mr. Weissman happy) that experience is also language dependent. It is unclear how dependent it is; for example, it appears that when children learn color words, how they see color begins to change (but this is obviously difficult to measure--link below). Anyway, language is not dependent on verbal/written communication (there are also hand gestures, intonations, etc.); but picking up on these other forms of communication is something that we learn. We have the capacity to do so, but meaning isn't derived from capacity. It is learned.
    –  Jon
    Oct 17, 2012 at 20:41
  • @Jon - Ah, yes.. science. Yet it is not certain if philosophy should be taking its cue from, say, physics or biology or psychology. On that topic, I tend to gravitate towards the ideas of A.N. Whitehead who considers philosophy to be an interpretative framework rather than a descriptive one. An excellent introduction to the philosophical part of Whitehead's work is Stubborn Fact and Creative Advance.
    –  Saul
    Oct 25, 2012 at 10:31
  • Sounds very much like the ontological argument for the existence of God to me.
    –  AndrewC
    Aug 20, 2014 at 15:19
  • @Saul, you wrote "Nothing is the opposite of Something, it does not and can not be"; then what do you suppose remains of your inner experience once you die? do you imply consciousness is eternal?
    –  nir
    Oct 11, 2014 at 18:45
  • @nir - The context here is metaphysics and not physics nor biology, hence the term "Something" does not and cannot refer to any particular and concrete entity. Instead, the term refers simultaneously to all real entities, processes and phenomena (both known and unknown) existing at any given moment. Something is eternal for sure, but when you go down to particular and individual stuff, then it becomes a completely different topic. For that, you should probably start with the elements and problems of Identity and then continue from there.
    –  Saul
    Oct 13, 2014 at 10:33
  • @Saul, can you consider consciousness as separate from the problem of (personal) identity? note for example that consciousness is not mentioned in the SEP article on identity; Lets discuss this in this room
    –  nir
    Oct 13, 2014 at 12:31
  • @nir - You are missing the whole point. This answer does not, cannot and shall not address anything less than the totality of everything there is. In regards to identity, it is not restricted to persons at all. Identity of any existent is that which enables the observer to identify and differentiate it from everything else. However, neither identity nor conciousness are the topics here. In order to discuss those things, you need to look somewhere else.
    –  Saul
    Oct 13, 2014 at 15:54
  • @Saul, out of curiosity - I can understand your comments if you view consciousness as a reducible phenomena which can emerge from the right form of computation; is that how you see it?
    –  nir
    Oct 13, 2014 at 16:52
  • @nir - Neither. I am saying that you are simply off-topic here. The eternal nature of Something does not place any requirements nor guarantees on the qualities of its potential constituent parts (including the conciousness of an individual man). It simply is what it is.
    –  Saul
    Oct 13, 2014 at 17:58
  • @Saul, I was actuallly trying to express an argument in the other direction - (a) we know that something (a universe) exists exactly because we are self aware (cogito ergo sum) and (b) if we cannot conceive our own inevitable nothingness, it could imply that nothing is a possible end for the universe as well, despite its inconceivability
    –  nir
    Oct 13, 2014 at 18:15
  • @nir - You are doing the same mistake as before, by incorrectly extrapolating the qualities of parts (the possibility of limits) onto the absolute totality of all there is. It is incorrect because absolute totality cannot have any external limits at all. If it could, then there should be other existents relative to it (as every limit requires at least two sides), but that would already be in contradiction with our initial premise (Something is all-encompassing). Hence, at least for Something, no possible ends exist. Not in the past, not in the present, and not in the future either.
    –  Saul
    Oct 13, 2014 at 20:07
  • What if we rephrase the question to: Why is there instead of isn't there? Sep 2, 2015 at 10:07
  • @YehudaShapira - The conclusion would be exactly the same because in the absolute sense "isn't there" simply cannot take the place of "is there". If it could, then it would immediately "be there", which is a contradiction to the initial premise that it isn't. Q.E.D.
    –  Saul
    Sep 3, 2015 at 15:54
  • "If it could, then it would immediately 'be there' " - would it? According to its definition, it wouldn't. Sep 6, 2015 at 7:27
  • @YehudaShapira - That's the point. Isn't there isn't there instead of is there because it neither wouldn't nor can't. The isn't there simply isn't there and the is there simply is there. Stated differently, you can put an apple to where an apple was previously missing but you cannot establish a space which lacks any boundaries to where none was before as such a space encompasses everything to begin with. That's why.
    –  Saul
    Sep 6, 2015 at 21:21
  • I don't think this can be argued. If you conclude "therefore there is not nothing", this implies "therefore there is something". but what something? you can't argue our something into existence. "nothing" in the original question should be read "not something". "why is there something rather than not something" or "why does something exist rather than not exist?" May 28, 2017 at 20:11
14

Definition: We say X is logically impossible if it entails a contradiction, and logically possible otherwise.

Definition: Given a set of assumptions, a sufficient explanation for X is a demonstration that not-X is logically impossible within those assumptions.

Lemma: If not-X is logically possible within a set of assumptions, then there does not exist a sufficient explanation for X (via reductio ad absurdum).

Assumption: An absence of affairs is logically possible.

Conclusion: There is no sufficient explanation for why there is something rather than nothing.

  • Thank you for your logical proof. Your conclusion would then refute the I think then the whole Universe is axiom, and underline the antagonism nature of the 2 discussed states. Your answer is then: We can't know. By the way, by absence of affairs you mean nothingness? Aug 23, 2011 at 17:33
  • 1
    @Geoffroy: Thanks, yes I mean nothing(ness), empty set, ontological void, what have you.
    –  anon
    Aug 23, 2011 at 17:42
  • But if there is a set of assumptions, there is something (namely that very set of assumptions) and thus there is not nothing. Therefore given any set of assumptions, you can conclude there is not nothing.
    –  celtschk
    Oct 21, 2012 at 13:43
  • 1
    @celtschk That sounds like magical thinking somehow. To investigate what an absence of affairs would entail, one does not postulate the existence of someone investigating that idea or holding that assumption (which would indeed be counterfactual to nothingness). Frankly, I feel absolute logic is transcendent in the sense that it does not "exist" in any particular world, because to say it "exists" is simply to say that it holds true or corresponds to possibility, which is not the same as our speaking of the "existence" of other things.
    –  anon
    Oct 21, 2012 at 16:45
  • (Sorry for digging up an old one) While it is agreeable that a statement like "absolute logic is transcendent of the non-logical" has a separate notion of existence, the assumption you state involves the concept of "affairs" or existence itself. That makes it more concrete than a statement in the domain of pure logic. Mar 14, 2015 at 22:45
  • @celtschk I think you are correct. However, note that your line of reasoning does not say anything about why there is something. So I think, therefore (I conclude) the Universe is is valid reasoning. However I think, which is why the Universe is is nothing more than unsupported solipsism.
    –  eMansipater
    Oct 25, 2015 at 9:21
13

This is ultimately a meaningless question in light of logical positivism. One cannot ask questions which have no translation to observations, and in this case, one cannot ask "what would it be like if there were nothing?" Nor can we ask "what is the relative probability of something rather than nothing?" other than in the trivial sense of 100% certainty of something, because here it is.

It is very easy to fool your brain into asking nonsense questions. These are question which cannot be translated into sense impressions, whose answers have no bearing on the observations. Examples of these questions:

  • Do the He atoms in the sun support the Republican or Democratic candidate for US president?
  • Where is the number 7 located?
  • How much does Mac OSX software weigh?

Questions of this sort come up in physics all the time, and physicists have become exceptionally good at smelling when a question is nonsense:

  • What is the precise position of an electron in the ground state of an H atom?
  • Does the object thrown into a black hole really cross the horizon, or just get smeared out on the surface?
  • Are quarks actually particles, or do the hadrons simply appear as if they are made out of quarks?
  • Which is the true value of the photon propagator Feynman's or Landau's ( $g_{\mu\nu}\over k^2$ or ${g_{\mu\nu} - {k_\mu k_\nu\over k^2} \over k^2$)?

These questions are completely ridiculous, physics doesn't answer them and doesn't admit that they need an answer. There are tons more, and you find as many as you like the more you delve into physics. This comes up right at the start, with the question "where is the coordinate zero in the Cartesian coordinates of the world located, really?" But these questions can fool the brain into thinking they are real questions, so physicists, starting with Ernst Mach, developed positivism to deal with them.

Positivism says, whenever you ask a question, ask yourself the following:

  • Assuming the answer is this ot that, is there any effect on empirical observation? Do any sensory impressions change depending on the answer?
  • Does any chain of reasoning depend on the assumption of the answer? Can this reasoning always be translated to another assumption?

If the answer is no and yes respectively, then the question is meaningless.

The principle of positivism moots many questions, including that of "existence". If I have an cup, and it keeps all its attributes (visibility, water-holding, hardness,etc) but loses the property of "existence", how does it change? It obviously doesn't. So I can imagine that the cup is winking in and out of existence, and this does nothing to the empirical relations, so it does nothing at all to sense impression, and the question of existence is meaningless.

Carnap noted that the positivism has the effect of culling away many traditional philosophical questions:

  • If the world is deterministic, how can we have free will?
  • Where did the universe come from?
  • Why is there something rather than nothing?

These and many others, essentially all of classical philosophy, are mooted or answered by applying logical positivist principles. This is codified in Carnap's book on the nonsensical nature of metaphysics. In the 1950s this was considered revolutionary, but by the 1970s, philosopers missed the old questions, so they quietly killed positivism and buried Carnap.

The notion of positivism is alive and well in physics, past quantum mecanics at least, but going back much further. The positivism is the source of all the influential philosophy and most of the influential physics of the 20th century, and it really is impossible to go back.

Your question is meaningless in light of positivism, and has no answer, nor does it need an answer.

  • I think there is a good point in here somewhere, Ron. Carnap and his positivist view of logical semantics is interesting, but the relevant aspect of his positivism for this question is his separation of Internal and External questions. Consider: Propositional logic does not require that we stipulate object domains other than the truth values of sentences. Why, then, should we prefer a model that actually does use objects? It seems like that's a matter of practical expressibility, and hence an External question of theory choice, rather than one Internal to any particular philosophical theory.
    –  Paul Ross
    Oct 29, 2012 at 16:47
  • 1
    @PaulRoss: I don't prefer a model that uses object domains, I have no problem with using predicate domains only, it is equivalent. You can make a formal language with one noun, "thing", and then my cup on the table is "thing of a white cuplike nature which is on the thing-supporting thing with four legs". You can omit thing by making it implicit and make a predicate only language. It really makes no difference, and one should accept and celebrate this freedom. Only the relationship between the truth values of formal sentences, given the translation from formalism to formalism, is invariant.
    –  Ron Maimon
    Oct 29, 2012 at 16:51
  • Sure; I agree that the freedom for alternative mathematical frameworks for theories is a great virtue. This external ability to play around with the formal underpinning of our theories is one of the key strengths of Carnap's philosophy. The point relative to this particular question is to note that the "meaninglessness" to which Carnap would attribute the OP's question of "why there is something rather than nothing" is an internal one. Since, qua Scientific Realism, we work in an internal mode, the question is dissolved; but relativising to the Internal mode is the crucial step.
    –  Paul Ross
    Oct 29, 2012 at 17:01
  • @PaulRoss: It took me forever to understand you, because you used the word "internal" in a way I didn't recognize. You mean "it is meaningless in and of itself" when you say "internal" above. That is, you can't assign meaning to this question without a whole bunch of metaphysical speculations. Then the "internal mode" just means you aren't allowing answers to questions that are internally meaningless. Well duh. If they're meaningless, they're meaningless, and there's no two ways about it. You don't need to relativise, there is no other mode. Carnap is just saying what Mach found self-evident.
    –  Ron Maimon
    Oct 30, 2012 at 5:26
  • Ron, that's not what I mean at all! I mean Internal in the sense of Carnap's "Empiricism, Semantics and Ontology" (1950) (I assumed you were familiar), shades of which also exist in Putnam's notion of Internal Realism; the idea that the questions are interpreted in terms of the framework of the theory, rather than as "meta" questions about such frameworks. "Being in itself" doesn't have much (if anything) to do with this idea; if anything, it's probably the example par excellence of External metaphysical theorising.
    –  Paul Ross
    Oct 30, 2012 at 14:35
  • @PaulRoss: The meaning Carnap gives to "internal" is not far from the one I ended up giving it off the cuff in the comments above. He means "meaningful internal to the system itself". I know, because I just read he article, and I also know, because I know the dude's philosophy, having reproduced it for myself as a young teenager (this is something all physicists do).
    –  Ron Maimon
    Oct 30, 2012 at 20:53
  • Hehehe. Fun. If science cannot reliably measure it, it's meaningless! Corollory: lust is meaningful but love is meaningless. I did particularly enjoy the meaninglessness of "Does the object thrown into a black hole really cross the horizon, or just get smeared out on the surface?".
    –  AndrewC
    Aug 20, 2014 at 15:15
  • First, science can measure love just as easily as lust, both are internal states that can be found by brain-probing, or much more easily reported by asking people "what are you feeling?". This is not about what science can measure, but rather about something more fundamental, what is the meaning one assigns to statements. The basic axiom is that the meaning is built upon the invariant content of peceptions, these can be verified directly. Then the frameworks built upon these must be consistent with these perceptions, and two frameworks which predict the same perceptions are really equivalent.
    –  Ron Maimon
    Aug 22, 2014 at 3:50
  • 2
    I agree with most of your answer, but why do you feel inclined to categorize "Are quarks actually particles, or do the hadrons simply appear as if they are made out of quarks?" as a nonsense question? (I'm a bit of a physicist too, and in particular, used to participate here.) Is it really in the same category as "Do the He atoms in the sun support the Republican or Democratic candidate for US president?", which is obvious nonsense. Former isn't that obvious to me. Can you clarify?
    –  299792458
    Aug 24, 2014 at 4:50
  • Because there was such an argument for a long time, about whether the definition of "particle" is for things you can separate, that have finite mass, or whether it means something that appears as a quantum of a renormalizable field theory. The latter definition won out, but it's simply a matter of convention, not a matter of fact, and as 't Hooft said "A particle is whatever is convenient for us to call a particle". So quarks became particles.
    –  Ron Maimon
    Aug 27, 2014 at 14:30
  • @RonMaimon - well, that's informative, so thanks. But even with this argument, the hadron could go around as a composite ''particle'', I mean it is certainly convenient ('t Hooft criterion). (Secondly,) the bone of contention is whether a hadron can viewed as a quantum of a renormalizable field theory. Please correct me if I'm wrong - this doesn't follow from QCD. It is possible to write effective Lags in low-energy QCD (I mean taking hadrons as the basic unit, rather than quarks, as in Chiral Perturbation Theory), but that ...
    –  299792458
    Aug 28, 2014 at 6:34
  • @RonMaimon - (contd.) ... doesn't make any hadron a quantum in the above sense. It is the effective degree of freedom, but not a quantum. Is that what you mean?
    –  299792458
    Aug 28, 2014 at 7:20
  • @New_new_newbie: If you make the proton fundamental, it isn't described by a renormalizable theory, it's magnetic moment is all wrong. What people decided is that a "particle" means an excitation of a field in a renormalizable theory, and that's a good enough definition for getting on. But yes, you can call the proton a "particle", or a "quantum" (it's always called a quantum). These questions are silly, it's just about terminology. You can call it whatever you want, the behavior in all experiments is understood.
    –  Ron Maimon
    Aug 31, 2014 at 17:02
  • @RonMaimon - Aha. I see. Thanks for the info. :)
    –  299792458
    Sep 1, 2014 at 4:46
10

Douglas Adams said:

A guy said to me, 'yes, but the whole theory of evolution is based on a tautology: that which survives, survives' This is tautological, therefore it doesn't mean anything.

I thought about that for a while and it finally occurred to me that a tautology is something that if it means nothing, not only that no information has gone into it but that no consequence has come out of it.

So, we may have accidentally stumbled upon the ultimate answer; it's the only thing, the only force, arguably the most powerful of which we are aware, which requires no other input, no other support from any other place, is self evident, hence tautological, but nevertheless astonishingly powerful in its effects. It's hard to find anything that corresponds to that and I therefore put it at the beginning of one of my books. I reduced it to what I thought were the bare essentials, which are very similar to the ones you came up with earlier, which were:

  • anything that happens, happens
  • anything that in happening causes something else to happen, causes something else to happen
  • anything that in happening causes itself to happen again, happens again

In fact you don't even need the second two because they flow from the first one, which is self-evident and there's nothing else you need to say; everything else flows from that.

So, I think we have in our grasp here a fundamental, ultimate truth, against which there is no gain-saying. It was spotted by the guy who said this is a tautology. Yes, it is, but it's a unique tautology in that it requires no information to go in but an infinite amount of information comes out of it. So I think that it is arguably therefore the prime cause of everything in the Universe. Big claim, but I feel I'm talking to a sympathetic audience.

To my mind what D.A. was getting at is that the origin-of-life question is the same as the origin-of-everything question: in both cases it seems counter-intuitive that stable systems can bootstrap themselves into existence. And that maybe these questions have the same answer: natural selection. Lee Smolin discussed his evolving universes idea with Richard Dawkins (who was a good friend of D.A.) here.

In both the origin-of-life and the origin-of-everything there were earlier proto forms that were very ropey and odd and inefficient and not what we think of as an actual universe/lifeform at all. Things like matter and indeed causality are highly-evolved end-products under this hypothesis.

  • 2
    My first thought about answering this question was just to write 42. but that was not long enough.
    –  Chad
    Jul 20, 2011 at 17:11
  • 14
    @Chad: "In the beginning the Universe was created. This has made a lot of people very angry and has been widely regarded as a bad move." Jul 20, 2011 at 17:45
  • 1
    The Universe might be the ultimate tautology, in more than one way. Example: Big Bang, stars appear, planets appear, life appears, life evolves, conscious beings appear and evolve, expand and reach extraordinary levels of knowledge and power, maybe involving something akin to a cosmic Singularity (as close to a secular concept of God as is possible), eventually learn to control space and time - and reach back in time all the way, triggering the Big Bang. In fact, I believe this is the simplest explanation of all. Jul 21, 2011 at 0:32
  • @Florin Andrei: In Greg Egan's novel Distress there's a physics conference which ends up creating the universe through such a tautology (though without the time travel).
    –  tim_hutton
    Jul 21, 2011 at 10:01
  • Calling the survival of the fittest a tautology is just wrong on so many levels. Most of them don't fit in a comment though, so just some food for thought: is "the fastest wins the race" a tautology?
    –  Tgr
    Jul 27, 2011 at 20:15
  • 3
    I do like the idea of evolving Universe. But in order to evolve, it needs some materials to begin with. With the "nothing" case, there is no materials at all. Aug 5, 2011 at 16:07
  • Steve Grand mentions in a blog entry that DNA "tried [this speech] out on Richard Dawkins and me the night before in the bar." Wow. :) stevegrand.wordpress.com/2010/10/10/101010
    –  tim_hutton
    Aug 22, 2011 at 11:01
  • @GeoffroyCALA: Yes, it's the same with natural selection: it seems hard for evolution to get started without a functional information-carrying replicator. But in both cases the answer is the same: there were earlier proto forms that were very ropey and odd and inefficient and not what we think of as an actual universe/lifeform at all. Things like matter and indeed causality are highly-evolved end-products under this hypothesis.
    –  tim_hutton
    Jul 7, 2014 at 14:44
  • It's not a tautology any more than the law of large numbers. One just has to distinguish between the ideal probability a given genotype has of surviving and reproducing (the fitness) that would occur in the limit of an infinite number of trials in the same type of environment (i.e. the true probability of surviving/reproducing as understood in frequentist terms), vs. the actual observed statistics in the finite number of real-world trials.
    –  Hypnosifl
    Aug 9, 2020 at 20:10
8

I think a typical answer to that question is the Anthropic Principle: if there was nothing instead of something, then we wouldn't be here to ask the question. So, even if the odds of something to nothing is one in a trillion, we're always going to be in the universe with something rather than the one with nothing. (This still leaves open the question as to why the odds wouldn't be zero in a trillion, however.)

Additional answers to this question (none of which are particularly satisfying), include:

  • An argument by Leibniz involving something similar to the Anthropic Principle combined with reductio ad absurdum
  • Why not?
  • The Anthropic principle is a valid answer but the problem is that it is answering it by avoiding it. But what if I really want to answer it? I would like a more precise and practical solution. Jul 20, 2011 at 11:27
  • 2
    @Geoffroy CALA: It isn't really avoiding it. There has to be something for you to ask the question. It is however pointing out that maybe it is The Wrong Question (tm). Reformulate it to "How did the universe come into existence" instead, and you get a better question. The answer still is "We have no idea, and we most likely will never have any idea". Jul 20, 2011 at 13:20
  • @Lennart Regebro Sorry, again you try to avoid my question, now you want to reformulate it! Jul 20, 2011 at 13:53
  • 4
    @Geoffroy CALA: No, I don't want to reformulate it, I'm just pointing out that you ask the wrong question. If you ask another one you might get a more useful answer. And that is not avoiding it. The question has been answered. You don't like the answer, but that's hardly my fault. Jul 20, 2011 at 19:00
  • 5
    -1: If you came home one day and saw your house floating twenty feet in the air, would you tell yourself: "This is because if it were otherwise, I wouldn't be seeing a floating house - but I am, so here it is." This is not an explanation! The question is not asking how we know there is something (which is what this is actually answering), it is asking why there is something rather than nothing.
    –  anon
    Aug 21, 2011 at 23:19
  • 1
    @anon: You're not understanding the anthropic principle. Whether house is floating does determine whether you can ask why your house is floating. Whether you exist determines uniquely whether you can ask why you exist. Consider this: the odds of getting 20 heads in a row are approximately one in a million. However, if ten million people flipped a coin 20 times, you would expect that several of them would get heads 20 times in a row. Now, if you only looked at the end result (not understanding how those people were selected), you'd be amazed that several people got 20 heads in a row.
    –  Ben Hocking
    Aug 22, 2011 at 21:52
  • 2
    I'm aware of the anthropic principle, @Ben, and it does not apply. There is no stochastic process determining whether there is something versus nothing, because that process itself would be something - hence there is no a priori reason to expect something to exist. You present a false analogy: an explanation demonstrates why X is the case rather than alternatives, whereas ten million people flipping coins will produce said alternatives en masse. The various runs of coin flips coexist as states of affairs, while "nothing exists" and "something exists" can't be simultaneously true.
    –  anon
    Aug 23, 2011 at 0:18
  • @anon: It is an analogy, not an equality. Not limited by characters, I'd add more about not being aware of the negative results. In our case, consider a potential infinite multiverse. In such a scenario there are an infinite unrealized universes and possibly only a finite (or singular) realized universe. However, that only addresses why "something exists" in our universe and not why the multiverse exists-a lack which is recognized in the original answer. I hope you can understand why I thought you misunderstood the anthropic principle based on your original house analogy.
    –  Ben Hocking
    Aug 23, 2011 at 11:11
  • Okay, I can understand that. The house analogy was only meant to outline what an explanation was; I didn't otherwise intend it as a comparable situation. (The original post said nothing about the multiverse explanation of fine-tuning or framing our single universe as only a single example in an ensemble, nor is my concern about that. Something and nothing are by definition mutually exclusive, unlike parallel/branched universes.)
    –  anon
    Aug 23, 2011 at 16:06
  • 1
    See the comments section of anon's answer: the anthropic principle is a reasonable argument for why we might observe probabilistically unlikely circumstances which are necessary for our existence, but it isn't an argument for why any circumstances, likely or unlikely, should exist at all. For that there can be no sufficient explanation.
    –  eMansipater
    Oct 25, 2015 at 9:27
  • @eMansipater, I do not disagree with anon's answer. It does have an assumption (hard to avoid), but it's a very reasonable assumption. I do not propose that my explanation is sufficient, merely that it is an explanation, of sorts. When I said, "none of which are particularly satisfying", I perhaps could've made it clear that includes the anthropic principle itself. However, while neither sufficient, nor particularly satisfying, I do think that the explanations I provide (as well as the other explanations here) are better than nothing (pardon the recursion).
    –  Ben Hocking
    Oct 25, 2015 at 14:52
  • I commented above that no-one had mentioned the Anthropic Principle and I realise now that my ability to scroll down leaves a lot to be desired... But yes, well put. I feel as though this is the one principle that many people do not thoroughly grasp, and it explains so much of not only deep scientific questions but deep philosophical questions.
    –  Sputnik
    May 30, 2018 at 15:14
8

What do you mean by "why"?

Do you mean "how did 'things' come to be?". That is a scientific question, and can be attacked by gathering and interpreting data (like what the physicists do).

If you mean, "what is a justification (a proof) that there is something (and not nothing)?", then the justification is that if there were nothing, you wouldn't be able to formulate such a question.

I think such a question is more figuratively motivated, a searching for psychological satisfaction in the consciousness of one's own consciousness, the existence of one's own thoughts. The answer to that, of resolving that tension, is to just get used to it.

  • 1
    By "how did things come to be", you assume there must have been a beginning, why? About the second reformulation, I don't need it because I already accept the axiom that something really exist by this reasonningm: I think then the Universe is. Finally, you're right that this question serve a purpose in some psychological effects it has on us. Some ignore the question, others wants to reformulate it, deny it, Jul 27, 2011 at 20:22
  • @Geoffroy: To point 1, I don't think there is that assumption, only that things do exist now (which you also accept) and that there is a scientific process for finding out -something- about how some things some into being if they do at all (possibly discovering that they -always- existed). Can you elaborate on what you intend by 'why' if it doesn't match well any of three I suggested?
    –  Mitch
    Jul 27, 2011 at 20:37
  • But I think this question leads to a world of valid interrogations and moreover, this question helps to answer several important questions; by imagining what could be a non-existent Universe, we realize how queer our actual Something is but only if we can subtract ourselves (humanity) from the equation. Jul 27, 2011 at 20:45
  • I am gonna try to be more explicit about the why: This is not a anthropocentric why so it is very close to your first "how did things come to be', just replace the past tense by present tense as with: How do things can be? Jul 27, 2011 at 20:52
5

It seems to me that the term nothingness can't even be defined. So, how can you say something is not nothing if you don't even know what nothing is.

But, as soon as you define nothing as a subset of U or partition any part of U to make room for nothing, then you have thereby defined nothing and defined it as separate from U. You then know what nothing is.

  • 2
    You are right when you say it's complicated to define both something and nothing. Maybe this is the key to solve or at least clarify this particular problem. Aug 14, 2011 at 19:59
4

Nothing is a word. That something exists can be verified by observation. Taking a word like existence (as in existence of everything), and then negating it, does not necessarily produce a meaningful concept. There is no meaning to the nonexistence of everything. In this case, it really makes no sense outside the context of our language's grammar and lexicon. It's just a word that refers to no observable facts (to NULL in a certain respect).

If you limit the context of the original concept of existence to a certain thing, or collection of things, then it becomes a statement of fact. That statement necessarily must have an inverse. If I state I wear a purple shirt, then either there is a purple shirt on my torso or there is not. So to say there does not exist that shirt is still a meaningful statement. One or the other statement is true, but not both. I cannot disentangle the word existence from meaningful facts without delving into absurdity. The moment you do that with any words, you end up with ridiculous statements that, though they may sound deep or illuminating, lack all meaning. They are just statements without any connection to the world outside of our own thoughts.

  • Yes, I understand that logically, the word nothing (in the context) is an absurdity. But does this means that nothingness (of the whole Universe) is definitely an impossibility, just because of a semantic argument? I mean, can we prove such a big fundamental physics statement (nothing does not exist) just with a semantic argument? Aug 11, 2011 at 11:54
  • By nothing does not exist I really meant nothingness could not exist instead of something Aug 23, 2011 at 17:44
3

This sort of question reminds me of an article I read about Einstein's theories on light (sorry, can't re-find the source). Back in early 19s, there were serious debates on light in order to understand if it was matter of if it was waves.

Newton's experimentations show light has matter-like properties but Max Planck's show wave-like properties. Who's right ? ... no-one known until Einstein came up with the hypothesis of light being both wave and matter.

Perhaps this situation is similare and the answer is : there are both present. Maybe there is a duality of both somethingness and emptyness whithin the same univers (or multivers). Our current model of reality tells us there is somthingness, but since human have a tendency to develop model-dependent realities (Stephen Hawking - The Grand Design), probabilities of "being right" with our current model of somethingness are very low, almost impossible.

In Breif, my answer to "why there is somethingness instead of nothing?" is "because somethingness fits better with our current understanding of univers" ... It will fit until a new model of reality comes up and enlarge our understanding to introduce emptyness in it. Those new models of reality are normally brought by fundamental scientific discoveries.

  • Thanks for your answer. Maybe you are right when you say they are both present, it's an idea I had already thought, but I could not be certain of its validity. My (debatable) idea was that maybe this question was invalid (as Louzer's answer) but only because our Universe is not in fact a TRUE something nor a TRUE nothing. Maybe a true something Universe would mean a infinitely dense Universe and a nothing Universe would mean a Zero density Universe. When our Universe is in fact something in between with a actual real (number) density. This notion is highly experimental! Aug 5, 2011 at 18:18
  • 3
    Nitpick: Part of your physics explanation is incorrect. You say that "Einstein came up with the hypothesis of light being both wave and matter." Well, not really. That's called "wave-particle duality" in physics circles, and Einstein rejected that theory. Louis de Broglie is probably better credited as the "inventor" of the wave-particle duality thesis. Aug 19, 2011 at 11:31
  • I am not a physics expert, so thanks for the note. The whole Einstein reference was more to point out that we trend to split concepts that can ultimatelly coexist. It's simply the use of an AND instead of a XOR in the philosophical-questionning process.
    –  ChrisEve
    Aug 24, 2011 at 14:39
3

My answer to the question of "Why is there something rather than nothing involves first providing a reason for why anything exists and then showing that what has traditionally been called "nothing" meets this reasoning and therefore really isn't "nothing" but actually exists.

In regard to the question of "Why do things exist?", I suggest that a thing exists if the contents of, or what is meant by, that thing are completely defined. A complete definition is equivalent to an edge or boundary defining what is contained within and giving "substance" and existence to the thing.

In regard to the question of "Why is there something rather than nothing?", "nothing", or "non-existence" is first defined to mean: no energy, matter, volume, space, time, thoughts, concepts, mathematical truths, etc.; and no minds to think about this lack-of-all. Next, I propose that this "non-existence" itself, and not our mind's conception of non-existence, in and of itself completely describes or defines the entirety of all that is present. Therefore, as a complete definition of what is present, what has traditionally been called "nothing", or "non-existence", is actually an existent state. That is, what has traditionally been called "nothing" is, when seen from a different perspective, an existent state or "something".

Another way to reach this same conclusion is by saying that in regard to the question of "Why is there something rather than nothing?", there are two choices for answering this question:

A. "Something" has always been here.

B. "Something" has not always been here.

Choice A is possible but does not explain anything. Therefore, choice B is the only choice with any explanatory power. With choice B, if "something" has not always been here, then "nothing" must have been here before it. "Nothing" is defined as above. But, in this complete "nothing, there would be no mechanism present to change this "nothingness" into the "something" that is here now. Because we can see that "something" is here now, the only possible choice then is that "nothing" and "something" are one and the same thing. This is logically required if we go with choice B.

Now, instead of saying "That can't be. Something and nothing are not the same", it's better to try and figure out how these two logically-required equivalent things can really be the same. The reason, I believe, is as described above that what has traditionally been called "nothing" completely describes, or defines, the entirety of what is present, and is therefore actually not "nothing", but is really an existent state.

Overall, I come to the conclusion that many others have come to which is that having true non-existence is not possible because even what we have traditionally thought of as true non-existence is really an existent state when seen from a different angle. Non-existence is basically a misnomer that has come about because we've been thinking of non-existence in our minds, which exist. Next to our existent minds, nothing just looks like nothing. But, nothing, or non-existence, itself, and not our mind's conception of non-existence, isn't dependent on being defined as the lack of existence. It's on its own and, on its own, completely describes the entirety of what is there and is thus really an existent state. In this (and all) areas, it's very important to distinguish between our mind's conception of non-existence and non-existence itself in which all minds along with everything else are gone.

This may all seem to be a waste of time, but the above reasoning provides a reason for why things exist and by trying to figure out the properties of the particular existent state previously referred to as non-existence allows one to build a model of the universe that is based on these properties via what I call "philosophical engineering". This model has symmetry breaking, a natural reason for why energy exists and a big bang like expansion of space.

A more thorough discussion of the above idea along with some responses to critiques is at my website at: https://sites.google.com/site/ralphthewebsite/filecabinet/why-things-exist-something-nothing

Also, some discussion of this has also taken place recently at google.com/groups in the epistemology and everything-list groups and many years ago at a digital philosophy group.

  • A rich answer in which I like several parts (but not all): When you say that nothing exists as well as something, I actually agree. And most of the answer is dedicated to define the 2 main concepts, that's good. Now sorry for the criticizing part: You say something may exist when it is completely defined (with boundaries). What about an infinite Universe with infinite mass and volume? Why do you totally ignore choice A "something has always been there", is it because your can't visualize/model/see it that your prefer the anthropomorphic birth view of Universe? Sep 4, 2011 at 14:59
  • I don't really understand your argument about admitting that the nothing existing as an 'existent state'. But then you say that 'true non-existence' is impossible, isn't it contradicting with your former argument for its existence? And in that part you also use our mind point of view to aid in your idea; well, I don't like using mind/consciousness tools to help formulate an argument, it's an anthropic type of answer which as value only in our limited lifetime, what about asking this question as if we never (humanity) existed? Sep 4, 2011 at 15:03
  • I do think the Universe has been there before the humans and will be there after our possible extinction, no need to include our subjective (naturally anthropomorphic) personal mind to the complex Universe equation. Finally, you make a reference to show how your paradigm fits well with the current Big-bang cosmology but that is supposing this model will never evolve and is right, (after millenaries of making Universe models, has humanity at last found the true model, well, that’s very improbable), moreover it is an ad-hoc reasoning. Sep 4, 2011 at 15:04
  • On a positive note, your answer has some very interesting parts (you arrive by different means by concluding that nothing exists as well as something) and we can tell you really thought of the problem; thank you for sharing your views with us. Sep 4, 2011 at 15:04
  • Hi. Thanks for your feedback! I may be doing this wrong (the "Add comment" box doesn't seem to allow much formatting in the form of white spacing, line breaks, etc.), but here are my responses: 1.) Check out my website on the infinity stuff because I've put stuff about infinite sets there. Basically, I'd say that based on my criterion of things existing if they are completely defined, that only actual infinites, and not not potential infinites, exist because what's contained within an actual infinite is completely defined but what's contained within a potential infinite is not.
    –  Roger
    Sep 5, 2011 at 3:17
  • Apparently, I can only edit a comment for 5 min. and have very few characters. Other responses are: 2.) The only reason I temporarily ignore choice A (something has always been here) is that, while it's possible, it offers no help in answering the question. So, choice B provides much more info. Then, in pursuing choice B and concluding that "nothing" and "something" are one and the same, this leads you back to A because it also means that "something" (in the form of what used to be called "nothing") has always been here. Y
    –  Roger
    Sep 5, 2011 at 3:20
  • 3.) When I say that true non-existence is impossible, I mean that because what we've traditionally called enon-existence is actually, when seen from a different perspective, an existent state, this means that non-existence that is not an existent state is impossible. I guess I wasn't so clear on that.
    –  Roger
    Sep 5, 2011 at 3:23
  • 1
    5.) I was purposely pointing out that it's important to NOT mix in our mind's conception of non-existence in trying to figure out what non-existence itself is like. One of the ways people think incorrectly about non-existence is to confuse our mind's conception of non-existence with non-existence itself. Next to our existent minds, nothing just looks like nothing. But, non-existence itself, not our mind's conception of it, isn't dependent on being defined as the lack of existence. It's on its own, and, on its own, completely defines the entirety of what is there and thus exists.
    –  Roger
    Sep 5, 2011 at 3:36
  • 1
    Response 5.) I didn't claim that my model wouldn't evolve. You read that into it. I'm constantly working on it, and when someone can give me logical reasons why it's not right or why another model is better, I'll go with that. But, given that, if one has faith in one's reasoning and no better reason has come along, it's perfectly warranted to think it's right, for now, and try to develop it further until it's proven wrong. So far, it seems to fit well with what I know of physics. It's good to have faith in one's reasoning and not trust too much to the value of group work, IMHO.
    –  Roger
    Sep 5, 2011 at 3:49
2

One additional interesting facet to consider, and apologies if I missed it in the earlier answers, comes from looking at the sum total of energy in the observable universe. As far as we've been able to measure so far, all the positive energy sources in the universe (positive electrical charges, mass, etc.) are exactly cancelled out by their equivalent negative energy source (negative electrical charges, gravity, etc.).

If the nature of the universe is such that the sum total of all energy actually IS exactly zero, we have the fascinating situation where the universe is, in a sense, 'nothing' - at least in the manner of:

1 - 1 + 2 - 2 + 3 - 3 = 0

Further support for this idea comes from physicists who are studying 'virtual particles'. It turns out that a pure vacuum is unstable, and 'virtual particles' can appear out of 'nothing' for a brief moment of time before re-combining and cancelling each other out again: a Nothing -> Something -> Nothing progression of the form: 0 = 1 - 1 = 0

In short, one of the unexpected answers to the ancient question is that, just maybe:

Σ(Universe) = 0

... Meaning once again, the universe appears to be turning our intuitions on their head: the 'something' that we see might actually be an expression of 'nothing'!

  • Interesting but 'energy' and 'vacuum' are both something by definition. And Zero energy (cancelled) doesn't mean 'nothing', it just mean that we can't currently fully comprehend the phenomenon with our limited tools. Aug 1, 2016 at 22:39
  • I am not saying we fully understand for instance positive or negative electric charges but at least we have more models for those. Aug 1, 2016 at 22:49
  • Our universe can be thought of us a pile of dirt and a hole. These are both "somethings". But put the dirt back in the hole and you get a nothing. In essence there is always nothing, there always was nothing. We just have the illusion of something. As said above, everything is energy in its fundamental form. Sum it all up and we get zero. By your logic nothing is also something .. so you are in a paradox.
    –  sci-guy
    Nov 15, 2016 at 22:30
2

Because nothing is a state of void being. This is an unnatural and unstable state. If you release something into a void it attempts to quickly disperse filling the void as much as possible. Similarly upon creation objects try to disperse. In a strange coincodence or maybe design things also attract each other. So while nature wants voids filled, matter wants to be together. As a result you end up with collections of particles scattered through out the universe.

Space is not truely empty. There are an effectively infintite number of atoms and particles out there. They are mostly dispersed so as to have an effective pressure approaching 0 through out the majority of the universe. But there are particles there. When they get close enough together and have enough attraction they couple. Eventually that couple collects more or becomes part of a larger collection.

So there is something because that is a more stable state. There are theories that there is antimatter collections in the universe that would cancel out most matter but the dispersal of matter/antimatter sometimes happens before the 2 can cancel each other out.

-- Edit addressing some Ben Hockings concerns

I propose that you can not sum space time. Thus there can be no totality of spacetime. That it may be infinite and immeasurable. That we indeed exist because that is a more natural state. That the universe as you have defined it expands to fill a limitless nothing because that is a more natural state than containing nothing.

Limitless is not the same as being infinitely large. It is nothing thus it has no size, and is not defined by our space time. No displacement is neccessary as there is nothing to displace. The universe expands to fill something that has no volume, or capacity thus can not be filled.

In some ways we agree that the void does not exist as part of our space time. But I deny that our spacetime contains the totality of all that is. The scientist in me would love to limit everything to the limits that transcribe. That would imply that the potential is there to understand everything even if the reality of that approaches the impossible. But logically that makes no sense. I can accept that we exist as statistical anomaly. But if we exist as a statisical anomoly of a cycle of creation/annialation what is the chance that that anomaly only exists once?

  • 1
    I agree, the OP has been operating under a false dichotomy since he was 10 years old (as have many Star Wars fans like me) that the opposite of the light is dark and the opposite of stuff is nothing.
    –  Peter Turner
    Jul 20, 2011 at 13:34
  • 1
    I'd argue that there is a difference between nothing and a void, although it is arguable. Not that it requires general relativity to argue, but since that's my reference frame (so to speak), I'd argue that a void still contains spacetime whereas "nothing" does not even contain that. As Stephen Hawking would argue it is an invalid question to ask what happened "before" the beginning of the Universe in the same manner that it is invalid to ask what is South of the South Pole.
    –  Ben Hocking
    Jul 20, 2011 at 13:47
  • @Ben Hocking - And I would argue that just because we require something to measure space time that does not mean that space time requires something to exist. Your premise requires a containment of infinty. What if the universe truely is limitless. You assume that because we can not measure, comprehend, or appreciate the concept of a truely limitless void that it can not exist. We often contstrain our universe to that which was created by the big bang. What if there are an infinite number of big bang type events.
    –  Chad
    Jul 20, 2011 at 14:08
  • @Ben Hocking I agree with you that nothing is not a void and it follows that Chad answer does not satisfy me. He already supposes existing matter dispersing into an unnatural void, but what if there was no matter, no space, no Universe? Chad also says that "Space is not truly empty" so then a true empty void would not even exist in our Universe, and it is also my opinion. My imagined "Nothing" does not exist in our Universe. Our know Universe is "something", OK. But why something exists at all? Jul 20, 2011 at 14:13
  • @Chad: actually, I'm not assuming that for the reasons you posit that I'm assuming that. I'm assuming that the spacetime of the universe is limited because that agrees with the predictions of General Relativity and observed measurements. (I.e., the combination of both of those, not either one by themselves.) General Relativity draws a distinction between nothing, a void of limited size, and a void of unlimited size.
    –  Ben Hocking
    Jul 20, 2011 at 14:26
  • A void is an object and void is used as a noun - in my statement "a state of void being" void is an adjective to the noun, being. Nothing has no bounds by definition because if it did then it would be something. The void I describe is nothing.
    –  Chad
    Jul 20, 2011 at 14:27
  • @Ben Hocking - General realitivity breaks down on the very small and vey large scales.
    –  Chad
    Jul 20, 2011 at 14:30
  • @Chad: General relativity does break down on very small scales, and quantum mechanics breaks down on large scales (I'm not aware of anyone saying that GR breaks down on large scales, however). That said, all proposed alternatives to GR that are seriously considered by the academic community maintain the distinctions that I refer to. Also, it should be said that some non-traditional solutions to GR do not necessarily break down on small scales. (For an example, do a Google scholar search with me as author and "alternative space time metrics".)
    –  Ben Hocking
    Jul 20, 2011 at 14:40
  • @Ben - By universe do you define that as all of existance including the area were it may be empty (Beyond the bounds of the expansion from the big bang) or just that which was created by the big bang?
    –  Chad
    Jul 20, 2011 at 15:18
  • @Chad - that's a little hard to answer without being explicit about the meaning of the word "empty". I use the word universe to refer to the totality of spacetime. There is no region "beyond" this in the same sense that one cannot go North of the North Pole. When saying that the universe is expanding, it's important not to say that it is expanding into anything. This spacetime, however, does not necessarily need to contain any non-virtual matter (although as you allude to in your answer, virtual particles will be popping in and out of existence in this region).
    –  Ben Hocking
    Jul 20, 2011 at 15:35
  • "If you release something" But then something exists. If nothing exists neither "you" or "something" exists, so you certainly can't release it. :-) Jul 20, 2011 at 19:02
  • That is a real world, observable, example. Not saying that the universe was released into a void. I havetried to clarify this.
    –  Chad
    Jul 20, 2011 at 19:37
  • @Chad: But it is an example that assumes that something exists in an effort to prove that something exists. Jul 20, 2011 at 20:33
  • @Lennert - Actually I use it to demonstrate a principle of dispersion and how it exists elsewhere. At least try to be accurate.
    –  Chad
    Jul 20, 2011 at 20:38
  • @Chad: I am being exact. You assume something exists, to argue that something exists. That is exactly what you are doing. I quote: "If you release something". QED. "Space is not truely empty." No, and space is also not "nothing". Space is something. QED again. "So there is something because that is a more stable state." Begging the question. You assume there is something to show that there is something. Jul 23, 2011 at 7:40
  • @Lennart - It is the principleof dispersion yes I assume space exists. The state of void being I am comparing to space. But that is NOT SPACE. I am talking about that which would not exist beyond the limits of existance. And referencing how dispersion works in existance to how the expansion of matter works. It is not the same but it is something we can reference to compare. I am not proving anyting I am positing a proposition. That is that it is more natural for matter to exist than for it not to exist.
    –  Chad
    Jul 25, 2011 at 13:20
  • @Chad: And when you argue that is more natural for matter to exist than for it not to exist, you do so by assuming that it exists. Also, the question was not about matter, but about something vs nothing. I can repeat this more times if that helps. You do assume something exists in your argument, and that begs the question. Jul 25, 2011 at 17:31
1

I'll be mostly repeating other good answers, but I'll be using different words, semantics and concepts.

There could never ever be nothing!

Because if there was nothing at all then, by definition, nothing could ever come out of it and we would never be able to be here, reasoning about it.

There, that's all.


Other than that...

We can go on and reason about the question in so many different ways. What I'll try to do below is bringing up some random reasoning to luckily connect a synapse in the OP's brain that makes sense. Because, in all truth, there is just nothing (instead of something) that can be said to answer the question as whole to everyone! :-)

For instance, back in 2005 I figured, for myself, the answer to "what's the reason of life, existence, and all things" to be there is no reason. To my neural network, that makes so much sense that the exact instant I came to this answer, with that exact wording, [I felt a formication in my brain (read the description, I've uploaded that video)][1] like never before or afterwards.

Back to the topic...

It can all be reduced to grouping things and defining words.

Let's define universe to everything that can possibly affect us somehow.

And let's define the nothing opposite to something as NOTHING, all in uppercase, in contrast to using semantics for talking about an empty group, the regular usage for nothing.

So, first, let's finally suppose you meant time (or space time) itself broader than the universe.

(1) spacetime > universe

Then it doesn't matter if there is NOTHING at any point.

NOTHING could have existed outside the universe group, at some point. Though it may also be the case that NOTHING never existed. It certainlly haven't existed inside our universe because if it did it would have affected us and there would be nothing instead of something. Since it's outside our universe, it can't matter, by definition. Nothing that exists outside of the universe matters to us.

But then we have two different groups (which might be the premise of your question): the NOTHING and the something, which includes the universe defined above.

If we were at the NOTHING group, we couldn't possibly be reasoning about it, by definition. So we are in the something group. There is something right now. There might be nothing at a place and time out of our universe - it can't matter.

Now let's go to the other possibility and assume...

(2) universe > spacetime

For whatever that means. Maybe the big bang created spacetime. Maybe there are things before the big bang which can affect us.

In any case, the universe includes some physical constraints that goes way and beyond anything we currently understand by reality.

Even then. If NOTHING was inside the universe group, it would be able to affect us. Again, what's a "NOTHING instead of something" which can affect us? It can only be, by definition something that would make it impossible anything to exist - or else there would be something.

Or worse the, NOTHING is inside the spacetime group. But wait, how can NOTHING even be inside any grouping?! If it exists anywhere alongside something it's alraedy not opposite to something!!

Ok... But wait! Why do we even exist then?!

That's a whole different question. The answer is: we have no fucking clue.

Then again, think of it this way: if there ever was an absolute NOTHING, how could anything come to exist? Answer is: there couldn't. That alone is already reason enough for something to exist. Else, NOTHING would be eternal and, well, what's the point in THAT?! ;P

The more you deeply reason about it

The more you can realize there could never, ever, be NOTHING.

  • Thanks for your answer: so there is both something and nothing at "same time in the same place", thus making the question invalid. As you said, you repeat stuff already said. So, I'll assume your grouping is of time greater than universe But you can't assume that because my definition of the whole Universe include everything, time included. Apr 14, 2012 at 12:22
  • And thanks for your hints, @GeoffroyCALA, on how I can improve my answer. I'm editing it now.
    –  cregox
    Apr 15, 2012 at 21:46
1

Since the empty set is also a set and therefore you have the whole set theory and nearly everthing logic requires. {} is the null statement and the empty set. Therefore there is a set and there is something. Here are more empty sets {<>,<>,<>} in fact 3 sets and there exists 3 things actually 4 things since the set of 3 sets is also a set and so on.

1

Asking "why" there is "something" (i.e., existence) is conceptually invalid: To ask "why" such-and-such a state of reality exists is implicitly to ask for a more detailed causal account of how things act to bring about this state. So if a person asks, "why is fire hot?" they are implicitly asking for a more detailed causal account of the heat of fire. A reasonable response to this might explain that fire is the oxidisation of certain materials, and explain why this physical process produces heat; a deeper explanation might further explain heat and temperature in physics terms and give a more detailed account of what "heat" actually is. One could ask an initial question like this, and then keep asking "why" at each explanation, and in each case, the respondent would attempt to provide a deeper account of the causal process, by going into more detail on the causal physical processes at work. What is important here is that asking "why" is a question about causality, and the answer is always to give a more detailed causal explanation.

So, when you ask "why is there something instead of nothing?", you are asking for a deeper causal account of the creation of "something" (i.e., a causal account of the existence of existence). This means that you are asking for a causal account for how "something" came to exist from a previous state that was not itself "something". Which means: you are asking for a causal account of how "something" is created from "nothing". Which means: you implicitly presume that something could not have been the start of things to begin with, and it must have sprung out of nothing. By the very nature of "something" (i.e., existence) there is nothing else outside of it, and so there is no thing or time outside of it. There is no preliminary state from which "something" could be the causal result. Hence there can be no causal account of the existence of "something" and so there is no "why".

(Incidentally, your question relates to a funny little experience that parents of young children have when little children incessantly ask "why". Parents of young children will be familiar with the fact that there is a limit to one's explanatory capacity when the child keeps asking "but why?" at each step of an explanation. This limitation represents the fact that deeper accounts of causality have a limit, though in the case of explaining things to a child this is more of a pedagogical limit than a metaphysical limit. Nevertheless, you can see that eventually there must be a starting point.)

1

The OP miscasts this famous question as physicalist rather than phenomenological.

Proceeding with a few quotes from the Stanford Encyclopedia of Philosophy : Nothingness

Let’s begin with a question that Martin Heidegger famously characterized as the most fundamental issue of philosophy. : Why is there something rather than nothing? . . .

‘Why is there something rather than nothing?’ is not about the origin of the world. Increasing the scientific respectability of the creation story (as with the Big Bang hypothesis) would still leave Heidegger objecting that the wrong question is being addressed.

Now switching to colourful quote from Camus, The Fall, (1956)

Please, let’s sit down. Well, what do you think of it? Isn’t it the most beautiful negative landscape? Just see on the left that pile of ashes they call a dune here, the gray dike on the right, the livid beach at our feet, and in front of us, the sea the color of a weak lye-solution with vast sky reflecting the colorless waters. A soggy hell, indeed! Everything horizontal, no relief; space is colorless, and life dead. Is it not universal obliteration, everlasting nothingness made visible? No human beings, above all, no human beings! You and I alone facing the planet at last deserted!

What makes everlasting nothingness visible is life, and what makes life is ('arguably') the self-organising principle and driving force of life: the Life Drive.

This concept of Life Drive is taking after Freud's Beyond the Pleasure Principal, Nietzsche's Will to Power, Heidegger's essence of being (Beyng), and Derrida's Life Drive in The Postcard, To Speculate--on "Freud", which I previously mentioned here.

So, not a question and answer about the Big Bang, but a more obvious question and answer about the originating principle of the beings who turn nothing into something.

1

No matter how one will want to answer this question, one would have to appeal, at least implicitly, to that which exists (or to what supposedly exists). Otherwise, one would put himself in the dubious position of assuming that the appeal to non-existence somehow explains existence. (The trend in philosophy since Plato, and perhaps long before him, is to posit some form of consciousness as the “answer” to such questions, even though this tactic is irrescindably incoherent.)

Thus, by posing this question and assuming that it is valid, you implicitly (but unavoidably) commits yourself to the fallacy of the stolen concept. If we ask why something is, but simply turn around and posit that something in our explanation of that something, what mileage have we gained? Indeed, we’re back to where we started, yet we don’t admit it to ourselves. This is what you do in assuming that the question “Why is there something rather than nothing?” (or “Why does existence exist?”) is a valid question. One will have to assume the fact of existence in order to answer the question. But in so doing, he will have to deny the fact of existence in order to validate his assumption that there must be a reason why there is something rather than nothing. He must assume the very concept his argument wants to deny, thus ‘stealing’ it from the objective hierarchy of knowledge, and rendering invalid any conclusion he hopes to draw from his argument.

Existence exists. We must start somewhere. The theist wants to start with a form of consciousness. He wants to posit a mind (albeit supernatural) which is responsible for creating all its objects. This is called metaphysical subjectivism, a view which holds that existence finds its source in a form of consciousness.

Some may object to my characterization of the question “Why is there something rather than nothing?” as fallacious, contesting that there is no such thing as a fallacious question. However, it is true when we examine issues in epistemology and logic, that there is a such thing as an invalid question. The fallacy known as ‘complex question‘, for instance, is a species of invalid question. It is a question which operates on a false assumption and expects the reader to accept that false assumption in order to answer it. The typical example is the question “Have you stopped beating your wife?” The question assumes that one is a married man and that he beats or has beaten his wife; indeed, it implies such beatings are a regular occurrence. Contrary to these assumptions, however, it could be the case a) that he is not married, or b) that he is married but has never beaten his wife. Since the question is asked in a manner in which a ‘yes’ or ‘no’ response can be the only appropriate reply, one cannot answer it on its own terms and avoid affirming its erroneous premises. One would implicate himself simply by answering. The question is fallacious because it leads one to accept a false premise, assuming either a) or b) are the actual case, if he should choose to take it seriously.

Likewise, a question which leads one to commit a fallacy in order to answer it is also invalid. If taken seriously, the question “Why is there something rather than nothing?” will lead one to commit the fallacy of the stolen concept; indeed, the fallacy of the stolen concept is unavoidable on the question’s own terms, as we saw above. One would have both to assume and deny existence in order to address the question. If Martin Heidegger ,who originally asked this question, did not recognize this, it was principally because he was not operating on a fully rational philosophy. Yet, today we have theists assuming this question is valid all the time in the construction of their apologetic ruses. What is it that theists want to posit in response to their invalid questions so as to appear to satisfy them? Of course, they assume that the only logical answer is to assert a universe-creating, reality-ruling form of consciousness, which they call God, and delight themselves with this as their answer, never allowing themselves to recognize that the question leads them to accepting a stolen concept, and assuming that their arguments justifying this illicit move make it valid.

  • 1
    Interesting answer. In short you say it is an invalid question, logically, but your argument didn't convince me. Could you be more specific here? "commits yourself to the fallacy of the stolen concept", what stolen concept? And I suppose the Universe already existed before my birth, so I don't need a human consciousness to prove it'll exist after my death (even if I proved the Universe does exist in the present thanks to my Consciousness, I take for granted it has/will forever existed/exist) Aug 3, 2011 at 17:00
  • When one or more concepts on which an argument logically depends are denied in the argument then that argument commits the fallacy of stolen concept. e.g. 1. There are absolutely no absolutely true statements. 2. Physics has proven science is incapable of telling us anything true. 3. It is impossible for people to communicate with one another. Aug 5, 2011 at 5:14
  • Yes. But these are statements, not questions. I can clearly see logical fallacies in your examples, but I see none in my question. Why natural chlorophyll is green instead of blue? we know natural chlorophyll is green, we know why it's green (cause on Earth surface green is the more efficient color to catch more energy from the Sun) so we also know why it isn't blue. No logical fallacies in the question. Aug 5, 2011 at 15:41
  • When you ask Why is there something instead of nothing, the ONLY answer you will accept is of the form: "There is something instead of nothing because there was <something> before there was something". Since you force the person who answers it to commit the fallacy of stolen concept, the question you asked is fallacious. I have explained it in detail above. Aug 5, 2011 at 16:39
  • 1
    Thank you. I now better understand your point. But the question remains valid for some class of answers as with notably Leo's answer: "There is something because nothing can't exist." or ChrisEve's "they are both present" The fallacy of the stolen concept doesn't seem to appy here. Aug 5, 2011 at 18:32
0

We don't really know. However since we can observe that there is something, rather than nothing, we can guess that it must have been harder for nothing to exist rather than something.

It's a guess and it need not be perfect, but I am afraid this might be as close we get to the answer of this question.

There is something rather than nothing, because it was easier for something to be.

0

Parmenides wrote in his poem, that he was brought before a goddess at the gates of Night and Day, in the abode of the Sun; and by steeds and chariot; and she told him there:

Come now, I will tell thee - and do thou hearken to my saying and carry it away - the only two ways of search that can be thought of. The first, that it is and that it is not possible for anything not to be - is the way of conviction

And continuing:

For truth is its companion; the other, namely that it is not - and that something must needs not be - that, I tell thee, is a wholly untrustworthy way. For you cannot know what is not - nor utter it.

Something then, always is; nothing - in the precise sense - can never be; even to name it, is to name something; in the imprecise sense - we have space, place, void, vacuums - but these, Parmenides is insisting is not nothing.

It suggests that Parmenides was reacting to, or deliberating, or compacting an argument of what it means to be and not, temporally or in-temporally.

Why is there something rather than nothing

Because the contrary - according to P - is precisely impossible; and is precisely unimaginable.

0

-Case 1: Nothing is not possible.

P_existence(nothing) = 0, P_existence(something) = 1. This answers the question.

-Case 2: Nothing is possible and it is the only possibility.

P_existence(nothing) = 1, P_existence(something) = 0. This is not our reality.

-Case 3: Nothing is possible. Transition probability to and from something is 0.

P_existence(nothing) > 0, P_existence(something) > 0.

Nothing is there, but we cannot experience it. The original question is under wrong premises: nothing is "somewhere", we just cannot experience it.

-Case 4: Nothing is possible. Transition probability to and from something are greater than zero.

Same as case 3, but we may transition to it in the future.

-Cases 5,6: As 3 and 4, but only one transition probability is zero.

I'm postulating the existence of a transition probability and an existence probability, which in a sense are more fundamental than even nothing and something. Case 4 is more natural, as it doesn't postulate measure zero probabilities. Too bad that there will be no experimenter around to test the prediction of a transition to nothing!

If optionally we want to address why we are in a "something state" since a long time ("long" in human subjective terms): perhaps the transition probability is asymmetric and much more likely in the nothing-->something direction, as in second law of thermodynamics. Indeed something has many microstates and large entropy, while the Boltzmann entropy of nothing is zero.

0

Despite Logical Positivists and others who will tell you that some questions are not fit to ask, I hold it as a personal truth that all questions that can be framed can be asked, and even from nonsense we can sometimes learn something, such as where we are personally mistaken in our understanding.

Given that, I say this looks like "what Martin Heidegger has called the fundamental question of metaphysics"as described by Art Witherall, so it has quite a pedigree! But its roots lie in the unresolved question of universals which dates back to Plato and Aristotle. See Problem of Universals on Wikipedia

One argument [given in answer to the question by @Saul] is: ".. if Nothing could be, then it would inevitably be Something - giving us a contradiction with our initial premises, thus demonstrating that Nothing cannot be."

But that has a whole bunch of issues about what kind of something, it seems on the face of it that it bootstraps Being, but really it just demonstrates the contradiction, not how the contradiction enables existence to be.

My understanding of the question is that the OP means how does it happen that we are here when so much is necessary to explain it? How does the Universe/Multiverse come about from an assumed earlier state of nothingness, or as an alternative to that state?

It is an expression of amazement, I think, and rightly so. And right up there with the top questions about us.

My answer is this:

I say there is a state I will call virtual that is very similar to the ideal of universals.

The nature of the virtual is that something of it is inherent in any reality [Axiom 1], at least, where it is created as a consequence of relationships. Therefore, if we consider a situation where no thing is real, nothing, no material reality, no time, no space, no God, we can see that this would be a real state of affairs, if it were so it would be a real null. So it would also imply a virtual state of affairs as the ideal of nothingness, which is the same as a null relationship. This is the something which the reality of nothing forces into being. It is not much, just a virtual link between nothing and itself.

It would be a dead end, going nowhere, if it were not for the second axiom concerning the virtual.

The other nature of the virtual is that whatever is logically implicit in one state [instance] of the virtual necessarily follows on from that state. [Axiom 2]

From axiom 2 we automatically, and necessarily, get the virtual state [predicate] of being. This virtual state is a relationship between the spirit [meaning/essence] of nothingness and the spirit [meaning/essence] of being. This is the first non-null relationship, but it is not a concrete thing, it is a virtual, abstract, thing, that exists nowhere and at no time.

That might not be perfectly argued, but the gist of it is, I think, valid.

The implications of this are many, but there are then even more steps to arrive at material existence. Which might include:

1) Number is a clear virtual consequence, and I imagine is still counting toward infinity, without being any closer, and in doing so provides a tick that is one measure of absolute time.

2) Dimension is a natural consequence of number, and so another virtual feature of existence, one that creates a multi-dimensional void.

3) Virtual time is only one thing happening after another, so having happened they remain, which is eternal.

4) Godel's Incompleteness Theorem suggests some things are unanswerable by mathematics alone, so there is a need for something like a universe to try things out and see what happens.

5) Consciousness could be in the relationships between the contents of mind, making it a virtual quality, rather than a physical quality of concrete brains.

6) The Big Bang takes place in the multi-dimensional void, creating a 3d Universe that expands as the surface with a common radius from ground zero. That surface is always now, and holds all the matter/energy. Consequentially yesterday is gone, and tomorrow not happened.

7) Material time is actually absolute, so all Relativity is about rate changing, not time. (space and time not really interchangeable, and no time travel)

8) Quantum effects are not interpretable as a block universe, but require something akin to the Transactional Interpretation to explain Young's Slits.

9) Other weird stuff happens, not yet accounted for by mainstream science.

10) Maybe death is not the end, except for our bodies.

So there you go. Maybe it is not all exactly correct, or is heretical in places, but that is the joy of asking questions, especially impossible ones!

PS. I am in no way qualified to claim any of the foregoing.

  • If you have any references to those who take similar views this would help support your answer and give the reader a place to go for more information. You mentioned Wittgenstein, Plato and Aristotle. Perhaps a quote from them would be useful. Welcome to Philosophy!
    –  Frank Hubeny
    Apr 20, 2019 at 22:21
  • Thank you @Frank. I have added links as suggested, and just as well because it was Heidegger I was thinking of, not Wittgenstein. As for the rest of it, I made it all up, so I don't know if anyone else ever thought the same thing. Personally I think universals are all relationships, and only sometimes between concrete particulars. I think Sir Roger Penrose also believes in the independent reality of number, but I'm not sure where I have seen that.
    –  Jeremy C
    Apr 21, 2019 at 9:14
0

As some answers have remarked: The term "nothing" does not denote an object. Instead it is the negation of a statement of existence. Hence I would reformulate the OP's question as:

Why is there a world?

My answer: We live in the world. If there were no world, we would not exist and could not ask this question.

That's a kind of anthropic principle. Continuing and discussing a possible cause of the world is outside the scope of current science.

0

It's simple:

  • If there were nothing, we could not ask the question.

  • We can ask the question, so there is not nothing.

  • As there is not nothing, there is something.

One could argue that this answer "plays tricks" with it's own meta level, but that is because the question itself does that.

0

Is quantum foam nothing? If not, did it arise spontaneously?

0

There might be nothing’ is false when read epistemically. (Roughly, a proposition is epistemically possible if it is consistent with everything that is known.) For we know that something actually exists and knowledge of actuality precludes all rival epistemic possibilities. But when read metaphysically, ‘There might be nothing’ seems true. So ‘Why is there something rather than nothing?’ is, so far, a live question... Knowledge, even a priori knowledge, that something is actually true [that something exists] is compatible with ignorance as to how it could be true.

If we grant that empty worlds are possible (the linked to article discusses that), then we probably can, I think, ask why this world isn't empty. We're quite used to asking how it is that something possible isn't actual: so why isn't the actual world empty?

I'm not sure there's any convincing explanation of why, because I can't think even of what sort of explanation could work.

Some philosophers conclude ‘Why is there something rather than nothing?’ is unanswerable. They think the question stumps us by imposing an impossible explanatory demand, namely, Deduce the existence of something without using any existential premises. Logicians should feel no more ashamed of their inability to perform this deduction than geometers should feel ashamed at being unable to square the circle.

But I also like this quote there, which suggests that our awareness of nothingness is important.

One need only shut oneself in a closet and begin to think of the fact of one’s being there, of one’s queer bodily shape in the darkness … of one’s fantastic character and all, to have the wonder steal over the detail as much as over the general fact of being, and to see that it is only familiarity that blunts it. Not only that anything should be, but that this very thing should be, is mysterious [William James]!

A good question without an answer, even if there's no consensus that it can't be answered: that would depend most on "explanatory standards" and what we're willing to accept as an 'explanation'.

0

The quesion assumes what it must prove. The view that nothing really exists or ever really happens is widepread and is crucial to Buddhist philosophy and mysticism in general. First one would have to show that something exists, before asking the question, and this cannot be done.

This would be why the quesion is unansswerable. It makes an incorrect asxumption. Or at least, the assumption cannot be shown to be correct. History shows that if you hang on to this assumption metaphyscis cannot be solved.

-1

A simple but fundamental answer:

There are only three answers to every question, "why?".

  1. "Why not?"
  2. "Because..."
  3. "You'll have to figure that out for yourself."

A note on answer #2: "because..." is ambiguous as the same answer can address two different questions very differently, e.g.

  • "Why are you shivering?" and
  • "Why are you putting on your sweater?"

...can both be answered by, "because I am cold."

In the former case, physical cause is cited, in the latter, logic.

Towards answer #3, consider that the question is imponderable and otherwise: know thyself.

-1

There is no equality at all in between "something" and "nothingness", and it asserts absolute differentiation (there is no the sameness at all) in between "something" and "nothingness".

Absolute differentiation in between "something" and "nothingness, it asserts essential differentiation (it’s not “i am different than you but we have the sameness a little bit”, but it’s strictly “there is nothing on me that exist within you”), therefore there is consequence:

  • If "something" has dimensional, then there will be no dimensional at all on another “nothingness”, and another “nothingness” is equal to “not exist” (we can ignore it, we consider it as “not exist”, since it requires no basic dimensional at all), and vice versa,

  • Whichever we choose, eventually it asserts there is only "dimensional", which asserts there is only "something" rather than "nothingness"

Why is there something instead of nothing?

  • Both "something" Versus "nothingness" asserts (consequence that there is) only "dimensional" (which left behind) that must be related to "something".

It may be considered incomparable in between "something" Versus "nothingness" (whether it maybe considered "doesn't make sense", contradiction in between both or whatever), but it can be considered as our trial to push our logical to the farthest extent and see where is it going to? And eventually it asserts there is only "dimensional" which assert "something".

Not the answer you're looking for? Browse other questions tagged or ask your own question.